Difference between revisions of "2016 AMC 10A Problems/Problem 7"

(Added elements from matching 12A problem)
(Solution)
Line 9: Line 9:
 
As <math>x</math> is the mean, <cmath>\begin{align*}
 
As <math>x</math> is the mean, <cmath>\begin{align*}
 
x=\frac{60+100+x+40+50+200+90}{7}
 
x=\frac{60+100+x+40+50+200+90}{7}
&\implies x=\frac{540+x}{7} \\
+
&\rightarrow x=\frac{540+x}{7} \\
&\implies 7x=540+x \\
+
&\rightarrow 7x=540+x \\
&\implies 6x=540 \\
+
&\rightarrow 6x=540 \\
&\implies x=\boxed{\textbf{(D) }90.}
+
&\rightarrow =\boxed{\textbf{(D) }90.}
 
\end{align*}</cmath>
 
\end{align*}</cmath>
 +
==Check==
 +
Order the list: <math>\{40,50,60,90,100,120\}</math>. <math>x</math> must be <math>60</math> or <math>90</math> because it is the median and mode of the set. Thus <math>90</math> is correct.
  
 
==See Also==
 
==See Also==

Revision as of 16:12, 4 February 2016

Problem

The mean, median, and mode of the $7$ data values $60, 100, x, 40, 50, 200, 90$ are all equal to $x$. What is the value of $x$?

$\textbf{(A)}\ 50 \qquad\textbf{(B)}\ 60 \qquad\textbf{(C)}\ 75 \qquad\textbf{(D)}\ 90 \qquad\textbf{(E)}\ 100$

Solution

As $x$ is the mean, \begin{align*} x=\frac{60+100+x+40+50+200+90}{7} &\rightarrow x=\frac{540+x}{7} \\ &\rightarrow 7x=540+x \\ &\rightarrow 6x=540 \\ &\rightarrow =\boxed{\textbf{(D) }90.} \end{align*}

Check

Order the list: $\{40,50,60,90,100,120\}$. $x$ must be $60$ or $90$ because it is the median and mode of the set. Thus $90$ is correct.

See Also

2016 AMC 10A (ProblemsAnswer KeyResources)
Preceded by
Problem 6
Followed by
Problem 8
1 2 3 4 5 6 7 8 9 10 11 12 13 14 15 16 17 18 19 20 21 22 23 24 25
All AMC 10 Problems and Solutions
2016 AMC 12A (ProblemsAnswer KeyResources)
Preceded by
Problem 3
Followed by
Problem 5
1 2 3 4 5 6 7 8 9 10 11 12 13 14 15 16 17 18 19 20 21 22 23 24 25
All AMC 12 Problems and Solutions

The problems on this page are copyrighted by the Mathematical Association of America's American Mathematics Competitions. AMC logo.png